can someone help me on this lr question? PT 30, sect 2, Q 21 Forum

Prepare for the LSAT or discuss it with others in this forum.
Post Reply
User avatar
flash21

Gold
Posts: 1536
Joined: Fri Apr 19, 2013 8:56 pm

can someone help me on this lr question? PT 30, sect 2, Q 21

Post by flash21 » Tue Oct 08, 2013 4:38 pm

the new agriculture bill will almost surely fail to pass. the leaders ...


I am not sure why A is correct opposed to E. I picked A but I got a bit lucky I think because I can't even see the difference between the two answer choices. I googled and found a forum of ppl talking about it but I'm still confused.. anyone know this one?

User avatar
Ambitious1

Bronze
Posts: 120
Joined: Wed Jul 11, 2012 10:38 pm

Re: can someone help me on this lr question? PT 30, sect 2, Q 21

Post by Ambitious1 » Tue Oct 08, 2013 5:19 pm

Conclusion: Bill will not pass. Why? Because leaders of all major parties said they oppose it.

Go into answer choices with an idea of the flaw. Just because leaders of major parties oppose it, does that mean that it will necessarily fail? Maybe the other party members outnumber the leaders.

The stimulus is assuming some sort of causal explanation between the bill passing/failing and what leaders think. So you can think of this as leaders oppose (cause) ---> bill not pass (effect)

(A) strengthens this causal connection. Most bills that have not been supported by even one member have not been passed into law. Pretty much restates oppose ---> bill not pass

(E) Shows bill pass ---> ~leaders oppose (negated)

If the conclusion were dealing with conditional reasoning, and not causal, (E) would strengthen the argument. However, the author isn't using a conditional statement to support his conclusion. He is assuming a causal connection between leaders opposing the bill and its effect on whether it gets passed or not.

allsystemsgo

New
Posts: 7
Joined: Sun Jan 27, 2013 7:29 am

Re: can someone help me on this lr question? PT 30, sect 2, Q 21

Post by allsystemsgo » Wed Oct 09, 2013 5:06 am

I thought of this one in terms of categorical logic because of the 'most' statements.

From A most> B, we can only derive B some> A. We don't know anything about ~B (or ~A for that matter).

This is why answer AC E is incorrect. It states, Bills passed into law MOST> supported by at least one leader. What inferences can we squeeze from this? Bill supported by at least one leader SOME> passed into law. That's it.

AC A, on the other hand, tells us: Bill not supported by one leader MOST> not passed into law. This is exactly what we want. If we take the input given in the stimulus "leaders all oppose it" and plug into our categorical statement, we get the output that it will 'most likely not pass into law'. And that is the conclusion we want to support.

I don't know why this page doesn't show up on a google search, but it's more helpful than other forums I've found. http://www.manhattanlsat.com/forums/q21 ... -t822.html

User avatar
flash21

Gold
Posts: 1536
Joined: Fri Apr 19, 2013 8:56 pm

Re: can someone help me on this lr question? PT 30, sect 2, Q 21

Post by flash21 » Wed Oct 09, 2013 7:23 am

i think the credited response is "A"

User avatar
Tetsuya0129

New
Posts: 1
Joined: Wed Mar 30, 2016 3:43 am

Re: can someone help me on this lr question? PT 30, sect 2, Q 21

Post by Tetsuya0129 » Thu Jan 03, 2019 9:29 am

Ambitious1 wrote:Conclusion: Bill will not pass. Why? Because leaders of all major parties said they oppose it.

Go into answer choices with an idea of the flaw. Just because leaders of major parties oppose it, does that mean that it will necessarily fail? Maybe the other party members outnumber the leaders.

The stimulus is assuming some sort of causal explanation between the bill passing/failing and what leaders think. So you can think of this as leaders oppose (cause) ---> bill not pass (effect)

(A) strengthens this causal connection. Most bills that have not been supported by even one member have not been passed into law. Pretty much restates oppose ---> bill not pass

(E) Shows bill pass ---> ~leaders oppose (negated)

If the conclusion were dealing with conditional reasoning, and not causal, (E) would strengthen the argument. However, the author isn't using a conditional statement to support his conclusion. He is assuming a causal connection between leaders opposing the bill and its effect on whether it gets passed or not.
Thanks, Ambitious1 for your great explanation. It helps a lot.

Want to continue reading?

Register now to search topics and post comments!

Absolutely FREE!


Post Reply

Return to “LSAT Prep and Discussion Forum”